The volume of a rectangular prism is 2 x cubed + 9 x squared minus 8 x minus 36 with height x + 2. Using synthetic division, what is the area of the base?
2 x cubed + 13 x squared + 18 x
2 x cubed + 5 x squared minus 18 x
2 x squared + 13 x + 18
2 x squared + 5 x minus 18

Answers

Answer 1

Answer:

2 x squared + 5 x minus 18

Step-by-step explanation:

Hope this helps sorry if not right

Answer 2

Answer:  D

Step-by-step explanation: EDGE


Related Questions

help meeeeeeeeee pleaseee !!!!!

Answers

The function 2x + 3x^2 represents the result of adding the two provided functions, f(x) and g(x).

Composite performance.

An operation known as "function composition" takes two functions, f and g, and produces a new function, h, that is equal to both g and f and has the property that h(x) = g.

Given the f(x) = 2x and g(x) = 3x^2 functions

The sum of the two functions must be calculated as illustrated;

f(x) + g = (f+g)(x)

Put the provided functions in place of (f+g)(x) to have:

(f+g)(x) = 2x + 3x^2

Standard version of the expression is (f+g)(x) = 2x + 3x^2

Consequently, the sum of the functions f(x) and g(x) is2x + 3x^2

Find out more about sums of functions here: https://brainly.com/question/17431959

#SPJ1

A soup can has a radius of 4.3 cm and a height of 11.6 cm. What is the volume of the soup can to the nearest tenth of a cubic centimeter?A. 1816.8B. 49.9C. 168.4D. 673.8

Answers

hello

to solve this problem, we need to identify the shape of the soup can first since soup is a liquid and carries the shape of whatever container its in.

volume of a cylinder is given as

[tex]\begin{gathered} V=\pi r^2h \\ \pi=3.142 \\ r=\text{radius} \\ h=\text{height} \end{gathered}[/tex][tex]\begin{gathered} v=\text{ ?} \\ r=4.3\operatorname{cm} \\ h=11.6\operatorname{cm} \\ \pi=3.142 \\ v=\pi r^2h \\ v=3.142\times4.3^2\times11.6 \\ v=673.9\operatorname{cm}^3 \end{gathered}[/tex]

from the calculations above, the volume of the soup is equal to 673.9cm^3 which corresponds with option D

Sarkis OganesyanCombine Like Terms (Basic, Decimals)May 20, 11:02:29 AMA triangle has side lengths of (1.1p + 9.59) centimeters, (4.5p - 5.2r)centimeters, and (5.3r + 5.4q) centimeters. Which expression represents theperimeter, in centimeters, of the triangle?14.89 + 5.6p + 0.2rO 0.1r + 5.6p + 14.99Submit Answer-0.7pr + 10.7qr + 10.6pq9.7qr + 10.9pr

Answers

The sides of the triagle have lengths:

1.1 p + 9.5 q

4.5 p - 5.2 r

5.3 r + 5.4 q

Or:

1.1 p + 0 r + 9.5 q

4.5 p - 5.2 r + 0 q

0 p + 5.3 r + 5.4 q

If we want to calculate the perimeter of the triangle, we just need to sum the three lenghts:

(1.1 + 4.5) p + (-5.2 + 5.3) r + (9.5 + 5.4) q

= 5.6 p + 0.1 r + 14.9 q

differentiate t^4 In(8cost)

Answers

⇒It is way more appropriate if I use the product rule. That states that:

⇒f(x)g(x)=f'(x)g(x)+f(x)g'(x)

[tex]t^{4} In(8cos(t))\\=4t^{3}In(8cos(t))+t^{4} \frac{1}{8cos(t)} *(0cos(t)+8*(-sin(t))*1)\\=4t^{3}In(8cos(t))+\frac{t^{4}-8sin(t)}{8cos(t)}[/tex]

Note:

Given F(x)=In(x)

⇒[tex]F'(x)=\frac{1}{x}[/tex]

Goodluck

Answer:

t^3 (4 ln(cos8t) - t tant)

Step-by-step explanation:

Using the Product Rule:

dy/dt = t^4 * d(ln(8cost) / dt + ln(8cost) * d(t^4)/dt

         = t^4 * 1/ (8cost) * (-8sint) + 4t^3 ln(8cost)

         = -8t^4 sint / 8 cost + 4t^3 ln(8cost)    

         = -t^4 tan t + 4t^3 ln(8cost)  

          = t^3 (4 ln(cos8t) - t tant)

7+[9÷(9x1 to the second power)]

Answers

The value of the expression 7+[9÷(9x1 to the second power)] is 64/9

What is a fraction?

A fraction can be described as the part of a whole set or element.

There are several types of fractions, which includes;

Simple fractionsComplex fractionsMixed fractionsProper fractionsImproper fractions

Some examples of these fractions are given as;

Simple fractions: 1/5, 1/6

Mixed fractions: 2 1/8, 3 1/4

Proper fractions: 2/3, 4/5

Improper fractions; 4/1, 6/3

Given the expression;

7+[9÷(9x1 to the second power)]

This is expressed as;

7 + ( 9 ÷ (9)^2

Find the square

7 + ( 9 ÷ 81)

find the ratio

7 + 1/9

Find the common multiple

63 + 1 /9

64/9

Hence, the value is 64/9

Learn more about fractions here:

https://brainly.com/question/11562149

#SPJ1

cos(alpha + beta) = cos^2 alpha - sin^2 beta

Answers

The trigonometric identity cos(α + β)cos(α - β) = cos²(α) - sin²(β) is verified in this answer.

Verifying the trigonometric identity

The identity is defined as follows:

cos(α + β)cos(α - β) = cos²(α) - sin²(β)

The cosine of the sum and the cosine of the subtraction identities are given as follows:

cos(α + β) = cos(α)cos(β) - sin(α)sin(β).cos(α - β) = cos(α)cos(β) + sin(α)sin(β).

Hence, the multiplication of these measures is given as follows:

cos(α + β)cos(α - β) = (cos(α)cos(β) - sin(α)sin(β))(cos(α)cos(β) + sin(α)sin(β))

Applying the subtraction of perfect squares, it is found that:

(cos(α)cos(β) - sin(α)sin(β))(cos(α)cos(β) + sin(α)sin(β)) = cos²(α)cos²(β) - sin²(α)sin²(β)

Then another identity is applied, as follows:

sin²(β) + cos²(β) = 1 -> cos²(β) = 1 - sin²(β).sin²(α) + cos²(α) = 1 -> sin²(α) = 1 - cos²(a).

Then the expression is:

cos²(α)cos²(β) - sin²(α)sin²(β) = cos²(α)(1 - sin²(β)) - (1 - cos²(a))sin²(β)

Applying the distributive property, the simplified expression is:

cos²(α) - sin²(β)

Which proves the identity.

Missing information

The complete identity is:

cos(α + β)cos(α - β) = cos²(α) - sin²(β)

More can be learned about trigonometric identities at https://brainly.com/question/7331447

#SPJ1

Sally started on the 12th floor. She walked up 4 flights. Then she went down 2 flights. Then she ran up 8 flights of stairs. a) Write an ADDITION expression b) What floor did she end up on? SHOW ALL WORK!

Answers

1) Gathering the data

Initial point 12th floor

2) She started on 12th floor and walked up 4 flights of stairs, assuming from each floor to another we have just 1 flight of stair. And we're using an addition expression, Hence, we can say:

12 +4-2+8=

16 +6

22

She ended up on the 22th floor

In windy cold weather, the increased rate of heat loss makes the temperature feel colder than the actual temperature. To describe an equivalent temperature that more closely matches how it “feels,” weather reports often give a windchill index, WCI. The WCI is a function of both the temperature F(in degrees Fahrenheit) and the wind speed v (in miles per hour). For wind speeds v between 4 and 45 miles per hour, the WCI is given by the formula(FORMULA SHOWN IN PHOTO)A) What is the WCI for a temperature of 10 F in a wind of 20 miles per hour?B) A weather forecaster claims that a wind of 36 miles per hour has resulted in a WCI of -50 F. What is the actual temperature to the nearest degree?

Answers

Let's remember what the variables mean:

F= temperature (in Fahrenheit),

v= wind speed.

A) The formula "works" when the wind speed is between 4 and 45 miles per hour. The question asks for a wind speed of 20 miles per hour. Then, we can apply the formula. Here,

[tex]\begin{cases}F=10 \\ v=20\end{cases}[/tex]

Then,

[tex]\begin{gathered} WCI(10,20)=91.4-\frac{(10.45+6.69\cdot\sqrt[]{20}-0.447\cdot20)(91.4-10)}{22}\approx\ldots \\ \ldots91.4-116.2857=-24.8857 \end{gathered}[/tex]

Approximating, the answer is

[tex]-25F[/tex]

B) This question is just about to find F in the provided equation after replacing the given v and WCI. Let's do that:

[tex]\begin{gathered} -50=91.4-\frac{(10.45+6.69\cdot\sqrt[]{36}-0.447\cdot36)(91.4-F)}{22}, \\ -141.4=-\frac{(10.45+6.69\cdot\sqrt[]{36}-0.447\cdot36)(91.4-F)}{22}, \\ -3110.8=-(10.45+6.69\cdot\sqrt[]{36}-0.447\cdot36)(91.4-F), \\ 3110.8=(10.45+6.69\cdot\sqrt[]{36}-0.447\cdot36)(91.4-F), \\ \frac{3110.8}{10.45+6.69\cdot\sqrt[]{36}-0.447\cdot36}=91.4-F, \\ F=91.4-\frac{3110.8}{10.45+6.69\cdot\sqrt[]{36}-0.447\cdot36}\approx1.2 \end{gathered}[/tex]

Then, the actual temperature is

[tex]1F[/tex]

Rewrite the following equation in slope-intercept form. x - 7y = 20 Write your answer using integers, proper fractions, and improper fractions in simplest form.

Answers

In this case, we'll have to carry out several steps to find the solution.

Step 01:

x - 7y = 20

slope-intercept form = ?

Step 02:

Slope-intercept form of the line

y = mx + b

x - 7y = 20

x = 20 + 7y

x - 20 = 7y

7y = x - 20

[tex]y\text{ = }\frac{x}{7}\text{ - }\frac{20}{7}[/tex]

The answer is:

y = x/7 - 20/7

What is 9207 /10 equivalent to?

Answers

Answer:

9207/10 is equivalent to 920.7

Use disks and washers to find the volume of the solid the results when the area of the region y=x^3 y = 0, and x = 2 is revolved about the line x= 2

Answers

Solution

The functions that define the region in consideration are given below:

[tex]\begin{gathered} y=x^3 \\ y=0 \\ x=2 \end{gathered}[/tex]

The Washer Method:

- Plotting these functions would help us visualize the question better. This is done below:

- The question would like us to revolve around the region about line x = 2. The region is bounded by the Blue, Red, and Green line. This requires that we use the formula given below:

[tex]\begin{gathered} V=\int ^b_a{f(y)\mathrm{dy}} \\ \text{where,} \\ a\text{ and }b\text{ are the bounds of the integration along the y-axis} \end{gathered}[/tex]

-

We can represent the region bounded by the function by rearranging the functions as follows:

[tex]undefined[/tex]

Hello, can you please help me solve this question ASAP!!!

Answers

SOLUTION:

Step 1:

In this question, we have that:

Step 2:

Part A:

We are meant to show that the equation:

[tex]5sinx=1+2cos^2x[/tex]

can be written in the form

[tex]2sin^2\text{x + 5 sin x - 3=0}[/tex]

Proof:

[tex]\begin{gathered} \text{5 sin x = 1 + 2 cos }^2x\text{ } \\ \text{But cos}^2x+sin^2x\text{ = 1} \\ \text{Then,} \\ \cos ^2x=1-sin^2x\text{ } \\ \text{Hence,} \\ 5sinx=1+2(1-sin^2x_{}) \\ 5sinx=1+2-2sin^2x \\ 5sinx=3-2sin^2x \end{gathered}[/tex]

Re-arranging, we have that:

[tex]2sin^2x\text{ + 5 sin x - 3 = 0 }[/tex]

Part B:

b) Hence, solve for x in the interval:

[tex]0\text{ }\leq\text{ x }\leq\text{ 2}\pi[/tex]

I’ve already done this problem, but I’m being told it’s wrong and I need to simplify but I don’t know how to do it with this question.

Answers

[tex]\begin{gathered} y=x^2-3 \\ For\text{ (1,2)} \\ x=1,\text{ y=2} \\ y=1^2-3 \\ y=1-3 \\ y=-2,\text{ -2}\ne2,\text{ hence} \\ \text{ this pair doesnt satisfy the equation }y=x^2-3 \\ \text{For (}4,13\text{)} \\ x=4,y=13 \\ y=4^2-3 \\ y=16-3 \\ y=13,\text{ 13=13, hence} \\ \text{This pair satisfies the equation }y=x^2-3 \\ \text{for (}-3,-9\text{)} \\ x=-3,\text{ y=-9} \\ y=(-3)^2-3 \\ y=9-3 \\ y=6,\text{ -9}\ne6,\text{ hence} \\ \text{ this pair doesnt satisfy the equation }y=x^2-3 \\ \text{For (}-5,22\text{)} \\ x=-5,\text{ y=22} \\ y=(-5)^2-3 \\ y=25-3 \\ y=22,\text{ 22=22, hence} \\ \text{This pair satisfies the equation }y=x^2-3 \end{gathered}[/tex]

Solve for x using the quadratic formula.3x^2 +10x+8=3

Answers

The quadartic equation is 3x^2+10x+8=3.

Simplify the quadratic equation to obtain the equation in standard form ax^2+bx+c=0.

[tex]\begin{gathered} 3x^2+10x+8=3 \\ 3x^2+10x+5=0 \end{gathered}[/tex]

The coefficent of x^2 is a=3, coefficient of x is b=10 and constant term is c=5.

The quadartic formula for the values of x is,

[tex]x=\frac{-b\pm\sqrt[]{b^2-4ac}}{2a}[/tex]

Substitute the values in the formula to obtain the value of x.

[tex]\begin{gathered} x=\frac{-10\pm\sqrt[]{(10)^2-4\cdot3\cdot5}}{2\cdot3} \\ =\frac{-10\pm\sqrt[]{100-60}}{6} \\ =\frac{-10\pm\sqrt[]{40}}{6} \\ =\frac{-10\pm2\sqrt[]{10}}{6} \\ =\frac{-5\pm\sqrt[]{10}}{3} \end{gathered}[/tex]

The value of x is,

[tex]\frac{-5\pm\sqrt[]{10}}{3}[/tex]

Please help me my answer is correct or no

Answers

Answer:

the answer is c actully

Step-by-step explanation:

iv'e took that test b4 so you welcome

A right triangle is shown in the graph.

right triangle on coordinate plane with hypotenuse labeled t and one endpoint of hypotenuse at r comma s and the other endpoint at x comma y, vertical line from point x comma y and horizontal line from r comma s that meet at right angle of triangle, horizontal dotted line from point r comma s to point s on y axis, horizontal dotted line from point x comma y to point y on y axis, vertical dotted line from point r comma s to point r on x axis, and vertical dotted line from right angle to point x on x axis


Part A: Use the Pythagorean Theorem to derive the standard equation of the circle with center at (r, s) and a point on the circle at (x, y). Show all necessary math work. (3 points)

Part B: If (r, s) = (7, –4) and t = 10, determine the domain and range of the circle. (4 points)

Part C: Is the point (9, 1) inside the border of the circle if (r, s) = (7, –4) and t = 10? Explain using mathematical evidence. (3 points

Answers

Part a: The standard equation of circle: (x - r)² + (y - s)² = t².

Part b: Domain = {17, -3} and Range = {-14, 6}.

Part c: Point (9, 1) lies inside the circle.

What is termed as the Pythagorean Theorem?The Pythagorean theorem, or Pythagorean theorem, explains the relation between the three sides of such a right-angled triangle. The the hypotenuse's square is equal to the total of the squares of the remaining two sides of a triangle, according to Pythagoras' theorem.

For the given question,

The right triangle are given with two of ts vertices as (r, s) and (x, y).

The distance between these two points is 't'.

Part a: The standard equation of the circle.

Centre of circle = (r,s) and

Point on the circle = (x, y)

Using Pythagorean Theorem,

(x - r)² + (y - s)² = t²

Thus, the standard equation of the circle is (x - r)² + (y - s)² = t²

Where, t is the radius of the circle.

Part b: Domain and range.

(r, s) = (7, –4) and t = 10,

For x values in the domain  r ± t and y values in the range s ± t, the circle would be defined.

Domain = 7 ± 10 = {17, -3}

Range = -4 ± 10 = {-14, 6}

Part c: Point (9, 1) lies inside or not.

(r, s) = (7, –4) and t = 10

Point (9, 1) = (x, y)

Put the values;

(x - r)² + (y - s)² ≤ t²

(9 - 7)² + (1 + 4)² ≤ 10²

2² + 5² ≤ 10²

4 + 25 ≤ 100

29 ≤ 100

Thus, the points (9, 1) lies inside the circle.

To know more about the Pythagorean Theorem, here

https://brainly.com/question/21332040

#SPJ13

Li’s family is saving money for their summer vacation. Their vacation savings account currently has a balance of $2,764. The family would like to have at least $5,000.Which inequality can be used to determine the amount of money the family still needs to save?

Answers

EXPLANATION

Savings account balance = $2,764

Desired amount = $5,000

Let's call x to the amount of money the family needs.

The inequality that could be used to determine the amount of money the family needs is the following:

2,764 + x ≥ 5,000

2) Katie and Jacob are enlarging pictures in a school yearbook on the copy machine. The ratio of the width to the length of the enlarged photo will be the same as the ratio of the width to the length of the original photo. 25 points One of the photographs that they want to enlarge is a 3" x 4"photo. katie says that she can enlarge the photo to a 9" x 12", but Jacob disagrees. He says it will be 11" x 12". Who is correct? Explain your reasoning in words. * Enlarged Photo Original Photo 3 inches 4 inches

Answers

The original picture Katie and Jacob want to enlarge is 3 by 4 photographs

This means that the initial length of the photograph is 3 and the intial width of the photographs is 4

If both of them want to enlarge the photograph, then the scaling factor must be the same for both the width and length

Katie enlarge the photo to a 9 x 12

The ratio of the original photograph is 3 to 4

That is, 3 : 4

Katie enlarge the photo to a 9 x 12

Ratio of the enlarged photo by katie is 9 to 12

That is, 9 : 12

Equate the two ratio together

3/4 = 9/12

Introduce cross multiplication

We have,

3 x 12 = 4 x 9

36 = 36

Therefore, the ratio which katie enlarged the photo results to a proportion

For Jacob

Jacob enlarged the photo to 11 x 12

Equating the two ratios

3/4 = 11/12

3 x 12 = 4 x 11

36 = 44

This does not give us a proportion

Therefore, Katie is correct while Jacob is wrong

If the price of gas was on average $2.85 per gallon, and thus was $1.36 cheaper than a year before, what is the percent of decrease in price?

Answers

The price of gas = $2.85 per gallon

It was $1.36 cheaper than a year before.

So, the price before = 2.85 + 1.36 = $4.21

So, the percent of decrease = 1.36/4.21 = 0.323 = 32.3%

1 mile= 1,760 yards.1 kilometer= 1,000 metersIf Jose walked 2 miles this morning, about how many kilometers did he walk?

Answers

1 mile= 1.609 km

Then,

2*1.609=3.218 km

He walked 3.218 kilometers

A straight line l1 with equation 5x - 7 = 0 cuts the x axis at point A. Straight line l2 is perpendicular to straight line l1 and passes through point A. What is the coordinates of point A and the equation of the straight line l2?

Answers

The coordinates of point A are (7/5, 0), and the perpendicular line that also passes through that point is:

y = 0.

How to get the perpendicular line?

Here we want to get a line perpendicular to:

5x - 7 = 0

Solving this for x, we get:

5x = 7

x = 7/5.

This is a vertical line, so the perpendicular line will be a horizontal line, which is of the form:

y = a.

We know that the line:

x = 7/5.

Cuts the x-axis at point A.

Remember that the x-axis as coordinates (x, 0).

So the coordinates of point A are (7/5, 0).

Now, the perpendicular line:

y = a

Needs to pass through the point (7/5, 0), so the value of a must be zero, then the line is:

y = 0.

Learn more about linear equations:

https://brainly.com/question/1884491

#SPJ1

Instructions: Fill in the table of values for the exponential function. Insert all answers as fractions, when applicable.

Answers

Given,

The expression is:

[tex]y=-2(\frac{1}{2})^x[/tex]

Required:

The value of y at x = -2, -1, 0, 1, 2.

The value of y at x = -2.

[tex]y=-2(\frac{1}{2})^{-2}=-2\times(2)^2=-2\times4=-8[/tex]

The value of y at x = -1.

[tex]y=-2(\frac{1}{2})^{-1}=-2\times(2)^1=-2\times2=-4[/tex]

The value of y at x = 0.

[tex]y=-2(\frac{1}{2})^0=-2\times(2)^0=-2\times1=-2[/tex]

The value of y at x = 1.

[tex]y=-2(\frac{1}{2})^1=-2\times\frac{1}{2}=-1[/tex]

The value of y at x = 2.

[tex]y=-2(\frac{1}{2})^2=-2\times\frac{1}{4}=-\frac{1}{2}=-0.5[/tex]

The table for the different value of the function:

x y

-2

How do I understand Standard Form of a Line? I don't know how to do it.

Answers

There are several forms in which one can write the equation of a line. Have in mind that TWO variables should be included in the equation. These two variables are: x and y.

If you type the equation in a form that looks like:

A x + B y = C

where the A, B, and C are actual numbers (like for example: 3 x - 2 y = 5)

This is the standard form of a line. to recognize it notice that bith variables x an y appear in separate terms on the LEFT of the equal sign., and a pure number (no variables) appears on the right of the equal sign.

Another form of writing the equation of a line is in the so called "solpe-intercept" form. This form looks like:

y = m x + b

Notice that in this case the variable ÿ" appears isolated on the left , and on the right of the equal sign you get a term with the variable x, and another constant (pure number) term (b). Like for example in the case of:

y = 3 x

Linear Programming WorksheetGraph each feasible region. maximize or minimize each objective

Answers

Given:

x+2y = 8

x=2, y=0

Substitute x=2 then find value of x as,

2+2y=8

2y=6

y=3

(x,y) = (0,3)

Now, substitute y=0 then find value of y as,

x+2(0)=8

x=8

(x,y) = (8,0)

It is given that P = x+3y

(x,y) = (0,3) then P= 0+3x3

P=9

The maximum valu P=9 and vertiex (0,3)

(x,y) = (8,0) then P=8+0= 8

The mininmum val

What is the remainder when 5x3 + 2x2 - 7 is divided by x + 9?-93,7503,800-3,490

Answers

Explanation

Given the expression

[tex]5x^3+2x^2-7[/tex]

The remainder when it is divided by x+9 can be seen below;

[tex]r=5(-9)^3+2(-9)^2-7=-3645+162-7=-3490[/tex]

Answer: -3490

need help asap look at attachment

Answers

Answer: Width =14, Length = 18

Step-by-step explanation:

L = W + 4

2W + 2L = 64

W+ L = 32

2W+ 4 = 32

2W = 28

W = 14

The with is 14 and length 18

Kiran is solving 2x-3/x-1=2/x(x-1) for x, and he uses these steps.He checks his answer and finds that it isn’t a solution to the original equation, so he writes “no solutions.” Unfortunately, Kiran made a mistake while solving. Find his error and calculate the actual solution(s).

Answers

Solution:

Given:

[tex]\begin{gathered} To\text{ solve,} \\ \frac{2x-3}{x-1}=\frac{2}{x(x-1)} \end{gathered}[/tex]

Kiran multiplied the left-hand side of the equation by (x-1) and multiplied the right-hand side of the equation by x(x-1).

That was where he made the mistake. He ought to have multiplied both sides with the same quantity (Lowest Common Denominator) so as not to change the actual value of the question.

Multiplying both sides by the same quantity does not change the real magnitude of the question.

The actual solution goes thus,

[tex]\begin{gathered} \frac{2x-3}{x-1}=\frac{2}{x(x-1)} \\ \text{Multiplying both sides of the equation by the LCD,} \\ \text{The LCD is x(x-1)} \\ x(x-1)(\frac{2x-3}{x-1})=x(x-1)(\frac{2}{x(x-1)}) \\ x(2x-3)=2 \\ \text{Expanding the bracket,} \\ 2x^2-3x=2 \\ \text{Collecting all the terms to one side to make it a quadratic equation,} \\ 2x^2-3x-2=0 \end{gathered}[/tex]

Solving the quadratic equation;

[tex]\begin{gathered} 2x^2-3x-2=0 \\ 2x^2-4x+x-2=0 \\ \text{Factorizing the equation,} \\ 2x(x-2)+1(x-2)=0 \\ (2x+1)(x-2)=0 \\ 2x+1=0 \\ 2x=0-1 \\ 2x=-1 \\ \text{Dividing both sides by 2,} \\ x=-\frac{1}{2} \\ \\ \\ OR \\ x-2=0 \\ x=0+2 \\ x=2 \end{gathered}[/tex]

Therefore, the actual solutions to the expression are;

[tex]\begin{gathered} x=-\frac{1}{2} \\ \\ OR \\ \\ x=2 \end{gathered}[/tex]

Suppose you want to have $ 749,791 for retirement in 13 years. Your account earns 9.5 % interest monthly. How much interest will you earn?$_________ (Round to the nearest DOLLAR)

Answers

ANSWER

$530,663

EXPLANATION

The amount the account will have in t years is given by,

[tex]A=P(1+\frac{r}{n})^{nt}[/tex]

Where n = 12, t = 13 years, r = 0.095 and A = 749,791. We have to find P,

[tex]P=\frac{A}{(1+\frac{r}{n})^{nt}}[/tex]

Replace with the values and solve,

[tex]P=\frac{749,791}{(1+\frac{0.095}{12})^{12\cdot13}}\approx219,128[/tex]

The interest earned is the difference between the initial deposit P and the final amount A,

[tex]i=A-P=749,791-219,128=530,663[/tex]

Hence, the interest earned would be $530,663.

what is 2 to the 6 power

Answers

[tex]\begin{gathered} 2\text{ to the power 6 is } \\ 2^6 \end{gathered}[/tex][tex]2^6=2\times2\times2\times2\times2\times2=64[/tex]

top question says: Triangle ABC can be taken to triangle A'B'C' using rigid motions and a dilation. help me pls

Answers

If triangle ABC can be taken to triangle A'B'C', it means that they are similar triangles. If tow triangles are similar, it means that the ratio of their corresponding sides are equal. Thus, we have

A'B'/AB = B'C'/BC = A'C'/AC

Thus, looking at the options, the true equations are

A) A'C'/B'A' = AC/BA

D) CA/C'A' = CB/C'B'

E) A'B'/AB = C'B'/CB

If we look at these options the ratios are always the same

Other Questions
Consider the functions below.Represent the interval where both functions are increasing on the number line provided. Select all the correct answers.Imagine that you have been asked to give a graduation speech. Which elements should you avoid when writing the speech?- a strictly formal, argumentative tone- the graduating students' accomplishments- funny quotes from other students- unpleasant remarks about a teacher- simple sentence constructions what is the process of providing a user with permission including access levels and abilities such as file access, hours of access, and amount of allocated storage space? Ashleys Internet service is terribly unreliable in fact on any given day theres a 60% chance that her Internets connection will be lost at some point that day what is the probability that her Internet service is not broken for seven days in a row inner a fraction or round your answer to four decimal places if necessary. if x=10 units, then what is the volume of the cube Select the graph for the solution of the open sentence. Click until the correct graph appears. Ix| + 3 > 3 The Lyon Restaurant Survey provides food, decor, and service ratings for some of the top restaurants across the United States. For 186 restaurants located in Boston, the average price of a dinner, including one drink and tip, was 48.60 Dollars. You are leaving on a business trip to Boston and will eat dinner 23 of these restaurants, randomly selected. Your company will reimburse you for a maximum of 50 dollars per dinner. Business associates familiar with these restaurants have told you that the meal cost at one-third of these restaurants will exceed 50 dollars.a. What is the probability that none of the meals will exceed the cost covered by your company?b. What is the probability that at most 12 of the meals will exceed the cost covered by your company? What is the probability that between 4 and 8 of the meals will exceed the cost covered by your company? c. Calculate the expected number of restaurants that will exceed the cost covered by your company. d. Calculate the probability of the first question by using the binomial distribution approximation. Therefore, in this case we will consider the possibility of repetition in the randomly selected restaurants. Define p=r/N as the success probability.N is the size of the population. r is the number of elements considered as successes in the population. e. Calculate the probability of the second question by using the binomial disribution approximation. f. Calculate the probability of the third question by using the binomial disribution approximation. g. Calculate the expected number of the fourth question by using the binomial disribution aproximation A washer and a dryer cost $765 combined. The washer costs $85 less than the dryer. What is the cost of the dryer? What is troubling about the circumstance when a coil is stationary and a magnet moves as compres to when a magnet is stationary and the coil moves? Find the pairs (LOTS OF POINTS!) look at the screenshots Object a attracts objects be the gravitational force of 10 N from a given distance the distance between the two objects is doubled what is the new force of attraction between If you select one card at random from a standard deck of 52 cards, what is the probability of that card being a 5, 6 OR 7? Mrs walters had a bag full of candy she wanted to share with 18 students. If she had 335 pieces of candy how many pieces will each student get pls help1. Which of the following was one of the keys to America's late 19th century industrial success?A. Shipping by seaB. Westward expansionC. The telegraphD. Railroads2. Which of the following natural resources helped support the production of the steel used for the America's railroads, machines, and cities?A. Iron oreB. GoldC. CoalD. Oil3. What was perfected by Henry Ford to help produce the Model T quickly and relatively cheaply?A. The assembly lineB. The combustion engineC. The electric engineD. The incandescent lightbulb4. Who is credited with the invention of the incandescent light bulb?A. Henry FordB. Alexander Graham BellC. Nikola TeslaD. Thomas Edison what events in donora, pennsylvania, and london, england, in the 1940s and 1950s helped spur the clean-air movement? responses construction of new high-polluting factories construction of new high-polluting factories deadly smogs deadly smogs mass protests mass protests passage of legislation in countries like cambodia or nigeria roads and driving patterns follow a more fluid pattern. this is an example of which cultural system? group of answer choices 50 points if this is correct i need this asap pleaseWhich is the best definition of a two-step equation?In a two-step equation, you must always start by using the division property of equality.In a two-step equation, you need to use one inverse operation to solve.A two-step equation is one that you need to use two properties of equality to solve.A two-step equation is one that you need to use only a single property of equality to solve. I need help graphing 3x+y=-1I already found the x intercept= -1/3 a is less than or equal to 10